LSAT and Law School Admissions Forum

Get expert LSAT preparation and law school admissions advice from PowerScore Test Preparation.

 rachue
  • Posts: 140
  • Joined: Jun 22, 2011
|
#1190
Hi, I'm trying to understand why D is correct over C. C seemed like it could be correct because it showed that the C and E needed by the stimulus' conclusion wasn't necessarily true. That seemed like it could be a flaw to me.
 Jon Denning
PowerScore Staff
  • PowerScore Staff
  • Posts: 904
  • Joined: Apr 11, 2011
|
#1372
D is correct by addressing the "anyone who follows our diet is sure to lose weight" idea: D says some people who followed that diet in the study group did not lost weight, so it shows the conclusion is too strong. C introduces the idea of an alternative, better way to lose weight (exercise), but since it does not address the diet information in the conclusion ("regardless of....") it is completely irrelevant to the argument.
 rachue
  • Posts: 140
  • Joined: Jun 22, 2011
|
#1492
That makes sense. Thanks!
 Nikki Siclunov
PowerScore Staff
  • PowerScore Staff
  • Posts: 1362
  • Joined: Aug 02, 2011
|
#19859
Here's a question that was sent to our attention earlier today:

October, 2008 section 1 Q 24

I originally answered D but then reread the stimulus. It didn't ask which choice would weaken the argument the most, it asked what possibility it overlooks. I just don't see any reason to assume this possibility has been overlooked. I have been doing really well on the LR lately and am just wondering if I'm overthinking things or maybe this question could've been asked better. I changed my answer to C because it offered an alternative explanation but I didn't like it either.

Thanks
 Nikki Siclunov
PowerScore Staff
  • PowerScore Staff
  • Posts: 1362
  • Joined: Aug 02, 2011
|
#19861
First, let me address your concerns regarding the question stem. Flaw and Weaken questions have a lot in common: if you can understand how to weaken the argument, then fundamentally you have some grasp of the flaw in it, and you can use that information to help you determine the correct answer choice. In other words, if you know a weakness in an argument, you are capable of seeing a concrete way to attack the argument, and this reveals, to some extent, the abstract nature of the flaw that is present.

To answer your specific question from PT55, Section I, Question 24, I'm inclined to treat this as a Weaken question, even though the stem is, technically, a Flaw stem ("the reasoning is most vulnerable to criticism on the grounds that the advertisement overlooks the possibility that..."). Why can I do this? Arguments frequently fail to take into account any number of different possibilities, most of which are entirely irrelevant to the logical validity of the conclusion. A discrete number of possibilities, however, are relevant and should have been taken into account because, if true, they could potentially weaken the argument.

Only possibilities that could potentially weaken the argument are possibilities that the author should have considered, and whose omission amounts to a logical flaw!

The advertisement recommends a diet which provides more calories from protein than from anything else, and which requires that breakfast be the biggest meal of the day. The author argues that anyone who follows this diet is sure to lose weight, because of a study showing that those who lost the most weight had a similar diet to the one being advertised. Let's call it the Atkins diet :)

This is a terrible line of reasoning. Just because the richest man in the world is in computer technology doesn't mean that anyone who get into computer technology is sure to become rich. Likewise, just because those who lost the most weight followed the Atkins diet doesn't mean that the diet will work for everyone. Maybe the "biggest losers" followed that diet, but how many of those who followed the Atkins diet actually lost weight? That's what I'd like to know... otherwise, imagine the following example:

You have 6 people (A, B, C, D, E, F) in a study comparing the effectiveness of the Atkins diet to that of the Mediterranean diet. 3 people were randomly assigned to each group. The results are as follows, with a designation of the weight each person lost:

Atkins:
A - 10 lbs
B - 0 lbs
C - 0 lbs

Mediterranean
D - 9 lbs
E - 9 lbs
F - 9 lbs

As you can see, the biggest loser followed the Atkins diet, suggesting that the Atkins diet can - in theory - cause one to lose more weight than the other diet. But which of the two diets is more likely to cause a random person to lose weight? Obviously, the Mediterranean - 100% of those who followed it lost some weight. By contrast, two of the three people on the Atkins lost no weight at all, suggesting that a random person following the Atkins diet is more likely not to lose weight (66%) than to lose weight (33%).

The advertisement clearly overlooks the importance of understanding what proportion of those who followed their diet lost weight, so that you can make an accurate prediction of your likelihood of losing weight by following their diet. The fact that the biggest loser followed the Atkins diet only means that the diet can potentially cause one to lose more weight than any other diet can, not that anyone is sure to lose weight following that diet.

Does that help you understand better why answer choice (D) describes a possibility that would weaken the argument?

Answer choice (C) would only be attractive if you misunderstood the language in the conclusion. The advertisement never promised that following the Atkins diet guarantees losing the most weight possible. Even if there were a way to lose more weight, the conclusion would still stand: it is still entirely possible that anyone who follows Atkins would lose weight, just not as much as they would lose if they also exercised.

Does that make sense? Let me know.

Thanks!

Get the most out of your LSAT Prep Plus subscription.

Analyze and track your performance with our Testing and Analytics Package.